Sei sulla pagina 1di 61

G.R. No. 163193, June 15, 2004 SIXTO S. BRILLANTES, JR., et.al, petitioner, VS. COMMISSION ON ELECTIONS, respondent.

(Digest by Ali Caronongan Sourced from class digest) FACTS : On December 22, 1997, Congress enacted Republic Act No. 8436 authorizing the COMELEC to use an automated election system (AES) for the process of voting, counting of votes and canvassing/consolidating the results of the national and local elections. It also required the COMELEC to acquire automated counting machines (ACMs), computer equipment, devices and materials and adopt new electoral forms and printing materials. The COMELEC initially intended to implement the said automation during the May 11, 1998 presidential elections, particularly in counting the votes collected from the Autonomous Region in Muslim Mindanao (ARMM). However, the failure of the machines to correctly read a number of automated ballots discontinued its implementation.

Contributions for the establishment of the AES persisted that even President Gloria Macapagal-Arroyo issued Executive Order No. 172 on January 24, 2003, allocating the sum of P2,500,000,000 to exclusively fund the AES in time for the May 10, 2004 elections. On February 10, 2003, upon the request of the COMELEC, President Gloria Macapagal-Arroyo issued Executive Order No. 175 authorizing the release of a further supplemental P500 million budget for the AES project of the COMELEC.

The Supreme Court resolved the COMELEC to maintain the old and manual voting and counting system for the May 10, 2004 elections after contract negations with companies Mega Pacific Consortium (the supplier of the computerized voting/counting machines) were discontinued. Despite this impediment, the COMELEC nevertheless continued the electronic transmission of advanced unofficial results of the 2004 elections for national, provincial and municipal positions, also dubbed as an "unofficial quick count."

ARGUMENTS:

Petitioner contends that the respondent COMELEC committed grave abuse of discretion amounting to excess of Jurisdiction in the issuance of Resolution No.

6712. Respondent COMELEC contends that its advancement in tabulation procedures is allowed within the statutory confines of section 52 (i) of the Omnibus Election Code that: Prescribe(s) the use or adoption of the latest technological and electronic devices, taking into account the situation prevailing in the area and the funds available for the purpose. Provided, That the Commission shall notify the authorized representatives of accredited political parties and candidates in areas affected by the use or adoption of technological and electronic devices not less than thirty days prior to the effectivity of the use of such devices. ISSUE: Whether or not Resolution No. 6712 dated April 28, 2004 issued by the COMELEC in authorizing the use of election funds in consolidating the election results for the May 10, 2004 elections should be declared VOID, as it is unconstitutional. HELD:

YES. For violating section 4 of Article VII. The said Resolution No. 6712 preempts the sole authority of the Congress to canvass the votes of the election returns for the President and the Vice-President.

REASONS: Art. VII, Sec. 4 of the 1987: Resolution Preempts the sole and exclusive authority vested in the Congress to canvass the votes for the election of President and VicePresident. It is a grave error on the part of the respondent to have ignored the misapprehensions addressed by Senate President Franklin M. Drilon to COMELEC Chairman Benjamin Abalos during the 2004 saying that such act would be in violation of the Constitution (section 4 of Article VII): "any quick count to be conducted by the Commission on said positions would in effect constitute a canvass of the votes of the President and Vice-President, which not only would be pre-emptive of the authority of Congress, but would also be lacking of any constitutional authority." The existence of an accredited Citizens arm: Under Section 27 of Rep. Act No. 7166, as amended by Rep. Act No. 8173, and reiterated in Section 18 of Rep. Act No. 8436, the accredited citizens arm - in this case, NAMFREL - is exclusively authorized to use a copy of the election returns in the conduct of an "unofficial" counting of the votes, whether for the national or the local elections. No other entity, including the

respondent COMELEC itself, is authorized to use a copy of the election returns for purposes of conducting an "unofficial" count. In addition, the second or third copy of the election returns, while required to be delivered to the COMELEC under the said laws, are not intended for undertaking an "unofficial" count. The said copies are archived and unsealed only when needed by to verify election results in connection with resolving election disputes that may be established. Inapplicability of Section 52(i) of the Omnibus Election Code: The Court contends that Section 52(i) of the Omnibus Election Code, which is cited by the COMELEC as the statutory basis for the assailed resolution, does not cover the use of the latest technological and election devices for "unofficial" tabulations of votes. Moreover, the COMELEC failed to notify the authorized representatives of accredited political parties and all candidates in areas affected by the use or adoption of technological and electronic devices not less than thirty days prior to the effectivity of the use of such devices, after failing to submit any document proving that it had notified all political parties of the intended adoption of Resolution No. 6712.

Macalintal vs PET, GR 191618, June 7, 2011

Posted by Pius Morados on November 13, 2011 (Admin Law, PET, Quasi-judicial power)

Facts: Par 7, Sec 4, Art VII of the 1987 Constitution provides: The Supreme Court, sitting en banc, shall be the sole judge of all contests relating to the election, returns, and qualifications of the President or Vice-President, and may promulgate its rules for the purpose.

Sec 12, Art. VIII of the Constitution provides: The Members of the Supreme Court and of other courts established by law shall not be designated to any agency performing quasi-judicial or administrative functions.

The case at bar is a motion for reconsideration filed by petitioner of the SCs decision dismissing the formers petition and declaring the establishment of the respondent PET as constitutional.

Petitioner argues that PET is unconstitutional on the ground that Sec 4, Art VII of the Constitution does not provide for the creation of the PET, and it violates Sec 12, Art VIII of the Constitution.

The Solicitor General maintains that the constitution of the PET is on firm footing on the basis of the grant of authority to the Supreme Court to be the sole judge of all election contests for the President or Vice-President under par 7, Sec 4, Art VII of the Constitution.

Issue:

Whether or not PET is unconstitutional. Whether or not PET exercises quasi-judicial power.

Held:

Yes. The explicit reference of the Members of the Constitutional Commission to a Presidential Electoral Tribunal, with Fr. Joaquin Bernas categorically declaring that in crafting the last paragraph of Sec. 4, Art VII of the 1987 Constitution, they constitutionalized what was statutory. Judicial power granted to the Supreme Court by the same Constitution is plenary. And under the doctrine of necessary implication, the additional jurisdiction bestowed by the last paragraph of Section 4, Article VII of the Constitution to decide presidential and vice-presidential elections contests includes the means necessary to carry it into effect. No. The traditional grant of judicial power is found in Section 1, Article VIII of the Constitution which provides that the power shall be vested in one Supreme Court and in such lower courts as may be established by law. The set up embodied in the Constitution and statutes characterize the resolution of electoral contests as essentially an exercise of judicial power. When the Supreme Court, as PET, resolves a presidential or vice-presidential election contest, it performs what is essentially a judicial power. The COMELEC, HRET and SET are not, strictly and literally speaking, courts of law. Although not courts of law, they are, nonetheless, empowered to resolve election contests which involve, in essence, an exercise of judicial power, because of the explicit constitutional empowerment found in Section 2(2), Article IX-C (for the COMELEC) and Section 17, Article VI (for the Senate and House Electoral Tribunals) of the Constitution.

Estrada vs. Desierto G.R. No. 146710-15, March 2, 2001 Estrada vs. Arroyo G.R. No. 146738, March 2, 2001 Sunday, January 25, 2009 Posted by Coffeeholic Writes Labels: Case Digests, Political Law

Facts: In the May 11, 1998 elections, petitioner Joseph Estrada was elected President while respondent Gloria Macapagal-Arroyo was elected Vice-President. From the beginning of his term, however, petitioner was plagued by problems that slowly eroded his popularity. On October 4, 2000, Ilocos Sur Governor Chavit Singson, a longtime friend of the petitioner, accused the petitioner, his family and friends of receiving millions of pesos from jueteng lords. The expose immediately ignited reactions of rage. On November 13, 2000, House Speaker Villar transmitted the Articles of Impeachment signed by 115 representatives or more than 1/3 of all the members of the House of Representatives to the Senate. On November 20, 2000, the Senate formally opened the impeachment trial of the petitioner. On January 16, 2001, by a vote of 11-10, the senator-judges ruled against the opening of the second envelope which allegedly contained evidence showing that petitioner held P3.3 billion in a secret bank account under the name Jose Velarde. The ruling was met by a spontaneous outburst of anger that hit the streets of the metropolis. Thereafter, the Armed Forces and the PNP withdrew their support to the Estrada government. Some Cabinet secretaries, undersecretaries, assistant secretaries and bureau chiefs resigned from their posts.

On January 20, 2001, at about 12 noon, Chief Justice Davide administered the oath to respondent Arroyo as President of the Philippines. On the same day, petitioner issued a press statement that he was leaving Malacanang Palace for the sake of peace and in order to begin the healing process of the nation. It also appeared that on the same day, he signed a letter stating that he was transmitting a declaration that he was unable to exercise the powers and duties of his office and that by operation of law and the Constitution, the Vice-President shall be the Acting President. A copy of the letter was sent to Speaker Fuentebella and Senate President Pimentel on the same day.

After his fall from the power, the petitioners legal problems appeared in clusters. Several cases previously filed against him in the Office of the Ombudsman were set in motion.

Issues: (1) Whether or not the petitioner resigned as President (2) Whether or not the petitioner is only temporarily unable to act as President

Held: Petitioner denies he resigned as President or that he suffers from a permanent disability.

Resignation is a factual question. In order to have a valid resignation, there must be an intent to resign and the intent must be coupled by acts of relinquishment. The validity of a resignation is not governed by any formal requirement as to form. It can be oral. It can be written. It can be express. It can be implied. As long as the resignation is clear, it must be given legal effect. In the cases at bar, the facts show that petitioner did not write any formal letter of resignation before leaving Malacanang Palace. Consequently, whether or not petitioner resigned has to be determined from his acts and omissions before, during and after Jan. 20, 2001 or by the totality of prior, contemporaneous and posterior facts and circumstantial evidence bearing a material relevance on the issue. The Court had an authoritative window on the state of mind of the petitioner provided by the diary of Executive Sec. Angara serialized in the Phil. Daily Inquirer. During the first stage of negotiation between Estrada and the opposition, the topic was already about a peaceful and orderly transfer of power. The resignation of the petitioner was implied. During the second round of negotiation, the resignation of the petitioner was again treated as a given fact. The only unsettled points at that time were the measures to be undertaken by the parties during and after the transition period. The Court held that the resignation of the petitioner cannot be doubted. It was confirmed by his leaving Malacanang. In the press release containing his final statement, (1) he acknowledged the oath-taking of the respondent as President of the Republic, but with the reservation about its legality; (2) he emphasized he was leaving the Palace, the seat of the presidency, for the sake of peace and in order to begin the healing process of the nation. He did not say he was leaving the Palace due to any kind of inability and that he was going to reassume the presidency as soon as the disability disappears; (3) he expressed his gratitude to the people for the opportunity to serve them; (4) he assured that he will not shirk from any future challenge that may come ahead in the same service of the country; and (5) he called on his supporters to join him in the promotion of a constructive national spirit of reconciliation and solidarity.

The Court also tackled the contention of the petitioner that he is merely temporarily unable to perform the powers and duties of the presidency, and hence is a President on leave. The inability claim is contained in the Jan. 20, 2001 letter of petitioner sent to Senate Pres. Pimentel and Speaker Fuentebella. Despite said letter, the House of Representatives passed a resolution supporting the assumption into office by Arroyo as President. The Senate also passed a resolution confirming the nomination of Guingona as Vice-President. Both houses of Congress have recognized respondent Arroyo as the President. Implicitly clear in that recognition is the premise that the inability of petitioner Estrada is no longer temporary. Congress has clearly rejected petitioners claim of inability. The Court cannot pass upon petitioners claim of inability to discharge the powers and duties of the presidency. The question is political in nature and addressed solely to Congress by constitutional fiat. It is a political issue which cannot be decided by the Court without transgressing the principle of separation of powers.

Neri vs Senate Committee on Accountability of Public Officers on September 22, 2011 00

Inquiry in aid of legislation Executive Privilege

Legislative (Sec 21) & Oversight (Sec 22) Powers

On 21 April 2007, DOTC entered into a contract with Zhong Xing Telecommunications Equipment (ZTE) for the supply of equipment and services for the National Broadband Network (NBN) Project in the amount of $329,481,290.00 (approximately P16 Billion Pesos). The Project was to be financed by the PRC. The Senate passed various resolutions relative to the NBN deal. On the other hand, De Venecia issued a statement that several high executive officials and power brokers were using their influence to push the approval of the NBN Project by the NEDA. Neri, the head of NEDA, was then invited to testify before the Senate Blue Ribbon. He appeared in one hearing wherein he was interrogated for 11 hrs and during which he admitted that Abalos of COMELEC tried to bribe him with P200M in exchange for his approval of the NBN project. He further narrated that he informed President Arroyo about the bribery attempt and that she instructed him not to accept the bribe. However, when probed further on what they discussed about the NBN Project, petitioner refused to answer, invoking executive privilege. In particular, he refused to answer the questions on (a) whether or not President Arroyo followed up the NBN Project, (b) whether or not she directed him to prioritize it, and (c) whether or not she directed him to approve. He later refused to attend the other hearings and Ermita sent a letter to the SRBC averring that the communications between GMA and Neri is privileged and that the jurisprudence laid down in Senate vs Ermita be applied. The SRBC cited Neri for contempt.

ISSUE: Whether or not the three questions sought by the SRBC to be answered falls under executive privilege.

HELD: The oversight function of Congress may be facilitated by compulsory process only to the extent that it is performed in pursuit of legislation.

The communications elicited by the three (3) questions are covered by the presidential communications privilege.

1st, the communications relate to a quintessential and non-delegable power of the President, i.e. the power to enter into an executive agreement with other countries. This authority of the President to enter into executive agreements without the concurrence of the Legislature has traditionally been recognized in Philippine jurisprudence.

2nd, the communications are received by a close advisor of the President. Under the operational proximity test, petitioner can be considered a close advisor, being a member of President Arroyos cabinet. And

3rd, there is no adequate showing of a compelling need that would justify the limitation of the privilege and of the unavailability of the information elsewhere by an appropriate investigating authority.

Civil Liberties Union VS. Executive Secretary

FACTS:

Petitioners: Ignacio P. Lacsina, Luis R. Mauricio, Antonio R. Quintos and Juan T. David for petitioners in 83896 and Juan T. David for petitioners in 83815. Both petitions were consolidated and are being resolved jointly as both seek a declaration of the unconstitutionality of Executive Order No. 284 issued by President Corazon C. Aquino on July 25, 1987.

Executive Order No. 284, according to the petitioners allows members of the Cabinet, their undersecretaries and assistant secretaries to hold other than government offices or positions in addition to their primary positions. The pertinent provisions of EO 284 is as follows:

Section 1: A cabinet member, undersecretary or assistant secretary or other appointive officials of the Executive Department may in addition to his primary position, hold not more than two positions in the government and government corporations and receive the corresponding compensation therefor.

Section 2: If they hold more positions more than what is required in section 1, they must relinquish the excess position in favor of the subordinate official who is next in rank, but in no case shall any official hold more than two positions other than his primary position.

Section 3: AT least 1/3 of the members of the boards of such corporation should either be a secretary, or undersecretary, or assistant secretary.

The petitioners are challenging EO 284s constitutionality because it adds exceptions to Section 13 of Article VII other than those provided in the constitution. According to the petitioners, the only exceptions against holding any other office or employment in government are those provided in the Constitution namely: 1. The Vice President may be appointed as a Member of the Cabinet under Section 3 par.2 of Article VII. 2. The secretary of justice is an ex-officio member of the Judicial and Bar Council by virtue of Sec. 8 of article VIII.

Issue:

Whether or not Executive Order No. 284 is constitutional.

Decision:

No. It is unconstitutional. Petition granted. Executive Order No. 284 was declared null and void.

Ratio:

In the light of the construction given to Section 13 of Article VII, Executive Order No. 284 is unconstitutional. By restricting the number of positions that Cabinet members, undersecretaries or assistant secretaries may hold in addition their primary position to not more that two positions in the government and government corporations, EO 284 actually allows them to hold multiple offices or employment in direct contravention of the express mandate of Sec. 13 of Article VII of the 1987 Constitution prohibiting them from doing so, unless otherwise provided in the 1987 Constitution itself.

The phrase unless otherwise provided in this constitution must be given a literal interpretation to refer only to those particular instances cited in the constitution itself: Sec. 3 Art VII and Sec. 8 Art. VIII.

Click this link! Check out our Updated List of Local Full-Time Jobs, Part-Time Jobs and Online Jobs. Case Digest on Estrada V. Arroyo Legitimacy of the Arroyo Presidency November 10, 2010

G.R. No. 146738 FACTS: Petitioner sought to enjoin the respondent Ombudsman from conducting any further proceedings in any criminal complaint that may be filed in his office, until after the term of petitioner as President is over and only if legally warranted. Erap also filed a Quo Warranto case, praying for judgment confirming petitioner to be the lawful and incumbent President of the Republic of the Philippines temporarily unable to discharge the duties of his office, and declaring respondent to have taken her oath as and to be holding the Office of the President, only in an acting capacity pursuant to the provisions of the Constitution. HELD: FIRST: The cases at bar pose legal and not political questions. The principal issues for resolution require the proper interpretation of certain provisions in the 1987 Constitution, notably section 1 of Article II, and section 8 of Article VII, and the allocation of governmental powers under section II of Article VII. The issues likewise call for a ruling on the scope of presidential immunity from suit. They also involve the correct calibration of the right of petitioner against prejudicial publicity. As early as the 1803 case of Marbury v. Madison, the doctrine has been laid down that it is emphatically the province and duty of the judicial department to say what the law is . . . The Court also distinguished between EDSA People Power I and EDSA People Power II. EDSA I involves the exercise of the people power of revolution which overthrew the whole government. EDSA II is an exercise of people power of freedom of speech and freedom of assembly to petition the government for redress of grievances which only affected the office of the President. EDSA I is extra constitutional and the legitimacy of the new government that resulted from it cannot be the subject of judicial review, but EDSA II is intra constitutional and the resignation of the sitting President that it caused and the succession of the Vice President as President are subject to judicial review. EDSA I presented political question; EDSA II involves legal questions.

SECOND: Using the totality test, the SC held that petitioner resigned as President. a. The proposal for a snap election for president in May where he would not be a candidate is an indicium that petitioner had intended to give up the presidency even at that time. b. The Angara diary shows that the President wanted only five-day period promised by Reyes, as well as to open the second envelop to clear his name. If the envelope is opened, on Monday, he says, he will leave by Monday. The President says. Pagod na pagod na ako. Ayoko na masyado nang masakit. Pagod na ako sa red tape, bureaucracy, intriga. (I am very tired. I dont want any more of this its too painful. Im tired of the red tape, the bureaucracy, the intrigue.) I just want to clear my name, then I will go. The SC held that this is high grade evidence that the petitioner has resigned. The intent to resign is clear when he said x x x Ayoko na masyado nang masakit. Ayoko na are words of resignation. c. During the negotiations, the resignation of the petitioner was treated as a given fact. The only unsettled points at that time were the measures to be undertaken by the parties during and after transition period. d. His resignation was also confirmed by his leaving Malacaang. In the press release containing his final statement, (1) he acknowledged the oath-taking of the respondent as President of the Republic albeit with the reservation about its legality; (2) he emphasized he was leaving the Palace, the seat of the presidency, for the sake of peace and in order to begin the healing process of our nation. He did not say he was leaving the Palace due to any kind of inability and he was going to reassume the presidency as soon as the disability disappears; (3) he expressed his gratitude to the people for the opportunity to serve them. Without doubt, he was referring to the past opportunity given him to serve the people as President; (4) he assured that he will not shirk from any future challenge that may come ahead in the same service of our country. Petitioners reference is to a future challenge after occupying the office of the president which he has given up; and (5) he called on his supporters to join him in the promotion of a constructive national spirit of reconciliation and solidarity. Certainly, the national spirit of reconciliation and solidarity could not be attained if he did not give up the presidency. The press release was petitioners valedictory, his final act of farewell. His presidency is now in the past tense. THIRD: The petitioner is permanently unable to act as President. Section 11 of Article VII provides that Congress has the ultimate authority under the Constitution to determine whether the President is incapable of performing his

functions. Both houses of Congress have recognized respondent Arroyo as the President. The House of Representative passed on January 24, 2001 House Resolution No. l75 which states: RESOLUTION EXPRESSING THE SUPPORT OF THE HOUSE OF REPRESENTATIVES TO THE ASSUMPTION INTO OFFICE BY VICE PRESIDENT GLORIA MACAPAGAL-ARROYO AS PRESIDENT OFTHE REPUBLIC OF THE PHILIPPINES, EXTENDING ITS CONGRATULATIONS AND EXPRESSING ITS SUPPORT FOR HER ADMINISTRATION AS A PARTNER IN THE ATTAINMENT OF THE NATIONS GOALS UNDER THE CONSTITUTION. The Senate also passed Senate Resolution No. 82 which states: RESOLUTION CONFIRMING PRESIDENT GLORIA MACAPAGAL-ARROYOS NOMINATION OF SEN. TEOFISTO T. GUINGONA, JR. AS VICE PRESIDENT OF THE REPUBLIC OF THE PHILIPPINES Implicitly clear in that recognition is the premise that the inability of petitioner Estrada is no longer temporary. Congress has clearly rejected petitioners claim of inability. Even if petitioner can prove that he did not resign, still, he cannot successfully claim that he is a President on leave on the ground that he is merely unable to govern temporarily. That claim has been laid to rest by Congress and the decision that respondent Arroyo is the de jure President made by a co-equal branch of government cannot be reviewed by the Supreme Court. FOURTH: The petitioner does not enjoy immunity from suit. The Supreme Court rejected petitioners argument that he cannot be prosecuted for the reason that he must first be convicted in the impeachment proceedings. The impeachment trial of petitioner Estrada was aborted by the walkout of the prosecutors and by the events that led to his loss of the presidency. On February 7, 2001, the Senate passed Senate Resolution No. 83 Recognizing that the Impeachment Court is Functus Officio. Since the Impeachment Court is now functus officio, it is untenable for petitioner to demand that he should first be impeached and then convicted before he can be prosecuted. The plea, if granted, would put a perpetual bar against his prosecution. The debates in the Constitutional Commission make it clear that when impeachment proceedings have become moot due to the resignation of the President, the proper criminal and civil cases may already be filed against him. The SC also ruled in In re: Saturnino Bermudez that incumbent Presidents are immune from suit or from being brought to court during the period of their incumbency and tenure but not beyond. Considering the peculiar circumstance that the impeachment process against the petitioner has been aborted and thereafter he lost the presidency, petitioner cannot demand as a condition sine qua non to his criminal prosecution before the Ombudsman that he be convicted in the impeachment proceedings. Also, petitioner cannot cite any decision of the SC licensing the President to commit criminal acts and wrapping him with post-tenure immunity from liability. The rule

is that unlawful acts of public officials are not acts of the State and the officer who acts illegally is not acting as such but stands in the same footing as any other trespasser. FIFTH: Petitioner was not denied the right to impartial trial. Pervasive publicity is not per se prejudicial to the right of an accused to fair trial. The mere fact that the trial of appellant was given a day-to-day, gavel-to-gavel coverage does not by itself prove that the publicity so permeated the mind of the trial judge and impaired his impartiality. In the case at bar, the records do not show that the trial judge developed actual bias against appellant as a consequence of the extensive media coverage of the pre-trial and trial of his case. The totality of circumstances of the case does not prove that the trial judge acquired a fixed opinion as a result of prejudicial publicity which is incapable if change even by evidence presented during the trial. Appellant has the burden to prove this actual bias and he has not discharged the burden.

FIRST DIVISION [G.R. No. 142283. February 6, 2003]

ROSA LIGAYA C. DOMINGO, ROMEO M. FERNANDEZ, VICTORIA S. ESTRADA, JULIETA C. FAJARDO, ADELAIDA B. GAWIRAN, MARCIANO M. SERVO, VICTORIA S. DAOANG, FELICIANO N. TOLEDO III, JAYNELYN D. FLORES, MA. LIZA B. LLOREN, ROMELIA A. CONTAPAY, MARIVIC B. TOLITOL, PAZ LEVITA G. VILLANUEVA, EDITHA C. HERNANDEZ, JOSE HERNANDEZ, JR., VERONICA C. BELLES, AMELITA S. BUCE, MERCELITA C. MARANAN, CRISTITUTO C. LLOREN, HERNANDO M. EVANGELISTA, and CARLOS BACAY, JR., petitioners, vs. HON. RONALDO D. ZAMORA, in his capacity as the Executive Secretary, HON. ANDREW B. GONZALES, in his capacity as the Secretary of Education, and HON. CARLOS D. TUASON, in his capacity as the Chairman of the Philippine Sports Commission, respondents. DECISION CARPIO, J.:

The Case

This is a petition for certiorari and prohibition[1] with prayer for temporary restraining order seeking to nullify Executive Order No. 81 and Memoranda Nos. 01592 and 01594.[2] The assailed executive order transferred the sports development programs and activities of the Department of Education, Culture and Sports (DECS for brevity) to the Philippine Sports Commission (PSC for brevity). The questioned memoranda (DECS Memoranda for brevity), on the other hand, reassigned all Bureau of Physical Education and School Sports (BPESS for brevity) personnel named in the DECS Memoranda to various offices within the DECS.

The Facts

On March 5, 1999, former President Joseph E. Estrada issued Executive Order No. 81[3] (EO 81 for brevity) entitled Transferring the Sports Programs and Activities of the Department of Education, Culture and Sports to the Philippine Sports Commission and Defining the Role of DECS in School-Based Sports.

EO 81 provided thus:

Section 1. Transferring the Sports Program and Activities to the PSC. All the functions, programs and activities of DECS related to sports development as provided for in Sec. 16 of EO 117 (s. 1987) are hereby transferred to PSC.

Section 2. Defining the Role of DECS in School-Based Sports. The DECS shall have jurisdiction and function over the enhancement of Physical Education (P.E.) curriculum and its application in whatever form inside schools.

Section 3. The Role of PSC. As the primary agency tasked to formulate policies and oversee the national sports development program, the management and implementation of all school-based sports competitions among schools at the district, provincial, regional, national and international levels, in coordination with concerned public and private entities shall be transferred to the PSC.

Pursuant to EO 81, former DECS Secretary Andrew B. Gonzales (Secretary Gonzales for brevity) issued Memorandum No. 01592 on January 10, 2000. Memorandum No. 01592 temporarily reassigned, in the exigency of the service, all remaining BPESS Staff to other divisions or bureaus of the DECS effective March 15, 2000.

On January 21, 2000, Secretary Gonzales issued Memorandum No. 01594 reassigning the BPESS staff named in the Memorandum to various offices within the DECS effective March 15, 2000. Petitioners were among the BPESS personnel affected by Memorandum No. 01594. Dissatisfied with their reassignment, petitioners filed the instant petition.

In their Petition, petitioners argue that EO 81 is void and unconstitutional for being an undue legislation by President Estrada. Petitioners maintain that the Presidents issuance of EO 81 violated the principle of separation of powers. Petitioners also challenge the DECS Memoranda for violating their right to security of tenure.

Petitioners seek to nullify EO 81 and the DECS Memoranda. Petitioners pray that this Court prohibit the PSC from performing functions related to school sports development. Petitioners further pray that, upon filing of the petition, this Court

issue a temporary restraining order against respondents to desist from implementing EO 81.

During the pendency of the case, Republic Act No. 9155 (RA 9155 for brevity), otherwise known as the Governance of Basic Education Act of 2001, was enacted on August 11, 2001. RA 9155 expressly abolished the BPESS and transferred the functions, programs and activities of the DECS relating to sports competition to the PSC. The pertinent provision thereof reads:

SEC. 9. Abolition of BPESS. All functions, programs and activities of the Department of Education related to sports competition shall be transferred to the Philippine Sports Commission (PSC). The Program for school sports and physical fitness shall remain part of the basic education curriculum.

The Bureau of Physical Education and School Sports (BPESS) is hereby abolished. The personnel of the BPESS, presently detailed with the PSC, are hereby transferred to the PSC without loss of rank, including the plantilla positions they occupy. All other BPESS personnel shall be retained by the Department.

The Issue

The issue to resolve is whether EO 81 and the DECS Memoranda are valid.

The Courts Ruling

We dismiss this petition for being moot and academic.

As manifested by both petitioners[4] and respondents,[5] the subsequent enactment of RA 9155 has rendered the issues in the present case moot and academic. Since RA 9155 abolished the BPESS and transferred the DECS functions relating to sports competition to the PSC, petitioners now admit that it is no longer plausible to raise any ultra vires assumption by the PSC of the functions of the BPESS.[6] Moreover, since RA 9155 provides that BPESS personnel not transferred

to the PSC shall be retained by the DECS, petitioners now accept that the law explicitly protects and preserves[7] their right to security of tenure.

Although the issue is already academic, its significance constrains the Court to point out that Executive Order No. 292 (EO 292 for brevity), otherwise known as the Administrative Code of 1987, expressly grants the President continuing authority to reorganize the Office of the President. Section 31 of EO 292 provides:

SEC. 31. Continuing Authority of the President to Reorganize his Office. The President, subject to the policy in the Executive Office and in order to achieve simplicity, economy and efficiency, shall have continuing authority to reorganize the administrative structure of the Office of the President. For this purpose, he may take any of the following actions:

(1) Restructure the internal organization of the Office of the President Proper, including the immediate Offices, the Presidential Special Assistants/Advisers System and the Common Support System, by abolishing, consolidating or merging units thereof or transferring functions from one unit to another;

(2) Transfer any function under the Office of the President to any other Department or Agency as well as transfer functions to the Office of the President from other Departments and Agencies; and

(3) Transfer any agency under the Office of the President to any other department or agency as well as transfer agencies to the Office of the President from other Departments or Agencies. (Emphasis supplied.)

Since EO 81 is based on the Presidents continuing authority under Section 31 (2) and (3) of EO 292,[8] EO 81 is a valid exercise of the Presidents delegated power to reorganize the Office of the President. The law grants the President this power in recognition of the recurring need of every President to reorganize his office to achieve simplicity, economy and efficiency. The Office of the President is the nerve center of the Executive Branch. To remain effective and efficient, the Office of the President must be capable of being shaped and reshaped by the President in the manner he deems fit to carry out his directives and policies. After all, the Office of the President is the command post of the President. This is the rationale behind the

Presidents continuing authority to reorganize the administrative structure of the Office of the President.

Petitioners contention that the DECS is not part of the Office of the President is immaterial. Under EO 292, the DECS is indisputably a Department of the Executive Branch. Even if the DECS is not part of the Office of the President, Section 31 (2) and (3) of EO 292 clearly authorizes the President to transfer any function or agency of the DECS to the Office of the President. Under its charter, the PSC is attached to the Office of the President.[9] Therefore, the President has the authority to transfer the functions, programs and activities of DECS related to sports development[10] to the PSC, making EO 81 a valid presidential issuance.

However, the Presidents power to reorganize the Office of the President under Section 31 (2) and (3) of EO 292 should be distinguished from his power to reorganize the Office of the President Proper. Under Section 31 (1) of EO 292, the President can reorganize the Office of the President Proper by abolishing, consolidating or merging units, or by transferring functions from one unit to another. In contrast, under Section 31 (2) and (3) of EO 292, the Presidents power to reorganize offices outside the Office of the President Proper but still within the Office of the President is limited to merely transferring functions or agencies from the Office of the President to Departments or Agencies, and vice versa.

This distinction is crucial as it affects the security of tenure of employees. The abolition of an office in good faith necessarily results in the employees cessation in office, but in such event there is no dismissal or separation because the office itself ceases to exist.[11] On the other hand, the transfer of functions or agencies does not result in the employees cessation in office because his office continues to exist although in another department, agency or office. In the instant case, the BPESS employees who were not transferred to PSC were at first temporarily, then later permanently reassigned to other offices of the DECS, ensuring their continued employment. At any rate, RA 9155 now mandates that these employees shall be retained by the Department.

WHEREFORE, the instant petition is DISMISSED. No pronouncement as to costs.

SO ORDERED.

Davide, Jr., C.J., (Chairman), Vitug and Azcuna, JJ., concur. Ynares-Santiago, J., no part.

Matibag vs. Benipayo, G.R. No. 149036, April 2, 2002

FACTS:

COMELEC en banc appointed petitioner as Acting Director IV of the EID. Such appointment was renewed in temporary capacity twice, first by Chairperson Demetrio and then by Commissioner Javier. Later, PGMA appointed, ad interim, Benipayo as COMELEC Chairman, and Borra and Tuason as COMELEC Commissioners, each for a term of 7 yrs. The three took their oaths of office and assumed their positions. However, since the Commission on Appointments did not act on said appointments, PGMA renewed the ad interim appointments.

ISSUES: Whether or not the assumption of office by Benipayo, Borra and Tuason on the basis of the ad interim appointments issued by the President amounts to a temporary appointment prohibited by Sec. 1(2), Art. IX-C Assuming that the first ad interim appointments and the first assumption of office by Benipayo, Borra and Tuason are legal, whether or not the renewal of their ad interim appointments and subsequent assumption of office to the same positions violate the prohibition on reappointment under Sec. 1(2), Art. IX-C

RULING:

Nature of an Ad Interim Appointment

An ad interim appointment is a permanent appointment because it takes effect immediately and can no longer be withdrawn by the President once the appointee has qualified into office. The fact that is subject to confirmation by the Commission on Appointments does not alter its permanent character. The Constitution itself makes an ad interim appointment permanent in character by making it effective until disapproved by the Commission on Appointments or until the next adjournment of Congress. The second paragraph of Sec.16, Art.VII of the Constitution provides as follows:

The President shall have the power to make appointments during the recess of the Congress, whether voluntary or compulsory, but such appointments shall be effective only until disapproval by the Commission on Appointments or until the next adjournment of the Congress.

Thus, the ad interim appointment remains effective until such disapproval or next adjournment, signifying that it can no longer be withdrawn or revoked by the President. xxx

...the term ad interim appointment means a permanent appointment made by the President in the meantime that Congress is in recess. It does not mean a temporary appointment that can be withdrawn or revoked at any time. The term, although not found in the text of the Constitution, has acquired a definite legal meaning under Philippine jurisprudence.

Rights of an Ad Interim Appointee

An ad interim appointee who has qualified and assumed office becomes at that moment a government employee and therefore part of the civil service. He enjoys the constitution protection that [n]o officer or employee in the civil service shall be removed or suspended except for cause provided by law. Thus, an ad interim appointment becomes complete and irrevocable once the appointee has qualified into office. The withdrawal or revocation of an ad interim appointment is possible only if it is communicated to the appointee before the moment he qualifies, and any withdrawal or revocation thereafter is tantamount to removal from office. Once an appointee has qualified, he acquires a legal right to the office which is protected not only by statute but also by the Constitution. He can only be removed for cause, after notice and hearing, consistent with the requirements of due process.

How Ad Interim Appointment is Terminated

An ad interim appointment can be terminated for two causes specified in the Constitution. The first cause is the disapproval of his ad interim appointment by the Commission on Appointments. The second cause is the adjournment of Congress without the Commission on Appointments acting on his appointment. These two causes are resolutory conditions expressly imposed by the Constitution on all ad interim appointments. These resolutory conditions constitute, in effect, a Sword of

Damocles over the heads of ad interim appointees. No one, however, can complain because it is the Constitution itself that places the Sword of Damocles over the heads of the ad interim appointees.

Ad Interim Appointment vs. Temporary Appointment

While an ad interim appointment is permanent and irrevocable except as provided by law, an appointment or designation in a temporary or acting capacity can be withdrawn or revoked at the pleasure of the appointing power. A temporary or acting appointee does not enjoy any security of tenure, no matter how briefly. This is the kind of appointment that the Constitution prohibits the President from making to the three independent constitutional commissions, including the COMELEC xxx

Was the renewal of appointment valid?

There is no dispute that an ad interim appointee disapproved by the Commission on Appointments can no longer be extended a new appointment. The disapproval is a final decision of the Commission on Appointments in the exercise of its checking power on the appointing authority of the President. The disapproval is a decision on the merits, being a refusal by the Commission on Appointments to give its consent after deliberating on the qualifications of the appointee. Since the Constitution does not provide for any appeal from such decision, the disapproval is final and binding on the appointee as well as on the appointing power. In this instance, the President can no longer renew the appointment not because of the constitutional prohibition on reappointment, but because of a final decision by the Commission on Appointments to withhold its consent to the appointment.

An ad interim appointment that is by-passed because of lack of time or failure of the Commission on Appointments to organize is another matter. A by-passed appointment is one that has not been finally acted upon on the merits by the Commission on Appointments at the close of the session of Congress. There is no final decision by the Commission on Appointments to give or withhold its consent to the appointment as required by the Constitution. Absent such decision, the President is free to renew the ad interim appointment of a by-passed appointee xxx

The prohibition on reappointment in Section 1 (2), Article IX-C of the Constitution applies neither to disapproved nor by-passed ad interim appointments. A disapproved ad interim appointment cannot be revived by another ad interim appointment because the disapproval is final under Section 16, Article VII of the Constitution, and not because a reappointment is prohibited under Section 1 (2), Article IX-C of the Constitution. A by-passed ad interim appointment can be revived by a new ad interim appointment because there is no final disapproval under Section 16, Article VII of the Constitution, and such new appointment will not result in the appointee serving beyond the fixed term of seven years.

Ulpiano Sarmiento III et al vs Salvador Mison & Carague on October 31, 2011

Political Law Appointments

This is the 1st major case under the 1987 Constitution. Mison was appointed as the Commissioner of the Bureau of Customs and Carague as the Secretary of the Department of Budget. Their appointment was done without the concurrence of the CoA. Ulpiano, being members of the bar, taxpayers, and professors of constitutional law questioned the appointment of the two sans confirmation by the CoA.

ISSUE: Whether or not the appointment is valid.

HELD: It is readily apparent that under the provisions of the 1987 Constitution, there are four (4) groups of officers whom the President shall appoint. These four (4) groups are:

First, the heads of the executive departments, ambassadors, other public ministers and consuls, officers of the armed forces from the rank of colonel or naval captain, and other officers whose appointments are vested in him in this Constitution;

Second, all other officers of the Government whose appointments are not otherwise provided for by law;

Third, those whom the President may be authorized by law to appoint;

Fourth, officers lower in rank whose appointments the Congress may by law vest in the President alone.

The 2nd, 3rd and 4th groups of officers are the present bone of contention. Should they be appointed by the President with or without the consent (confirmation) of the CoA? By following the accepted rule in constitutional and statutory construction that an express enumeration of subjects excludes others not enumerated, it would follow that only those appointments to positions expressly stated in the first group require the consent (confirmation) of the CoA.

Because of the conflicting extremes provided in the 2 previous Constitutions, the framers of the 1987 Constitution and the people adopting it, struck a middle ground by requiring the consent (confirmation) of the CoA for the 1st group of appointments and leaving to the President, without such confirmation, the appointment of other officers, i.e., those in the 2nd and 3rd groups as well as those in the 4th group, i.e., officers of lower rank.

As to the 4th group of officers whom the President can appoint, it was pointed out by the intervener CoA that the 3r sentence in Sec. 16, Article 7 of the 1987 Constitution, which reads:

The Congress may, by law, vest the appointment of other officers lower in rank in the President alone, in the courts, or in the heads of departments, agencies, commissions, or boards. since a law is needed to vest the appointment of lowerranked officers in the President alone, this implies that, in the absence of such a law, lower-ranked officers have to be appointed by the President subject to confirmation by the CoA; and, if this is so, as to lower-ranked officers, it follows that higherranked officers should be appointed by the President, subject also to confirmation by the CoA. Ulpiano et al argued that the third sentence of Sec. 16, Article 7, merely declares that, as to lower-ranked officers, the Congress may by law vest their appointment in the President, in the courts, or in the heads of the various departments, agencies, commissions, or boards in the government. No reason however is submitted for the use of the word alone in said third sentence

The SC ruled that both arguments are not correct. After a careful study of the deliberations of the 1986 Constitutional Commission, that the use of the word alone after the word President in said third sentence of Sec. 16, Article 7 is, more than anything else, a slip or lapse in draftsmanship.

In the 1987 Constitution the clear and expressed intent of its framers was to exclude presidential appointments from confirmation by the CoA, except appointments to

offices expressly mentioned in the first sentence of Sec. 16, Article 7. Consequently, there was no reason to use in the third sentence of Sec. 16, Article 7 the word alone after the word President in providing that Congress may by law vest the appointment of lower-ranked officers in the President alone, or in the courts, or in the heads of departments, because the power to appoint officers whom he (the President) may be authorized by law to appoint is already vested in the President, without need of confirmation by the CoA, in the second sentence of the same Sec. 16, Article 7.

Therefore, the 3rd sentence of Sec. 16, Article 7 could have stated merely that, in the case of lower-ranked officers, the Congress may by law vest their appointment in the President, in the courts, or in the heads of various departments of the government. In short, the word alone in the third sentence of Sec. 16, Article 7 of the 1987 Constitution, as a literal import from the 1935 Constitution, appears to be redundant in the light of the second sentence of Sec. 16, Article 7. And, this redundancy cannot prevail over the clear and positive intent of the framers of the 1987 Constitution that presidential appointments, except those mentioned in the first sentence of Sec. 16, Article 7, are not subject to confirmation by the CoA. Misons and Caragues appointments are affirmed affirmed.

Aquilino Pimentel vs Executive Secretary Ermita on October 31, 2011

Political Law Ad Interim Appointments

While Congress was in session, GMA appointed Arthur Yap et al as secretaries of their respective departments. They were appointed in acting capacities only. Pimentel together w/ 7 other senators filed a complaint against the appointment of Yap et al. During pendency, Congress adjourned and GMA re-issued ad interim appointments re-appointing those previously appointed in acting capacity. Pimentel argues that GMA should not have appointed Yap et al as acting secretaries because in case of a vacancy in the Office of a Secretary, it is only an Undersecretary who can be designated as Acting Secretary. Pimentel further asserts that while Congress is in session, there can be no appointments, whether regular or acting, to a vacant position of an office needing confirmation by the CoA, without first having obtained its consent; GMA cannot issue appointments in an acting capacity to department secretaries while Congress is in session because the law does not give the President such power.

ISSUE: Whether or not the appointments made by ex PGMA is valid.

HELD: Ermita, in behalf of the other respondents, argued that GMA is allowed under Sec. 16, Art 7 of the Constitution to make such appointments. Pursuant to the Constitution, the President shall have the power to make appointments during the recess of the Congress, whether voluntary or compulsory, but such appointments shall be effective only until disapproval by the CoA or until the next adjournment of the Congress. Ermita also pointed out EO 292 which allows such an appointment with the exception that such temporary designation shall not exceed one year. Sec 17, Chap 5, Title I, Book III of EO 292 states that [t]he President may temporarily designate an officer already in the government service or any other competent person to perform the functions of an office in the executive branch. Thus, the President may even appoint in an acting capacity a person not yet in the government service, as long as the President deems that person competent. Also, Congress, through a law, cannot impose on the President the obligation to appoint automatically the undersecretary as her temporary alter ego. An alter ego, whether temporary or permanent, holds a position of great trust and confidence.

Congress, in the guise of prescribing qualifications to an office, cannot impose on the President who her alter ego should be.

What Bernas Says

Ad-interim appointments must be distinguished from appointments in an acting capacity. Both of them are effective upon acceptance. But ad-interim appointments are extended only during a recess of Congress, whereas acting appointments may be extended any time there is a vacancy. Moreover ad-interim appointments are submitted to the Commission on Appointments for confirmation or rejection; acting appointments are not submitted to the Commission on Appointments. Acting appointments are a way of temporarily filling important offices but, if abused, they can also be a way of circumventing the need for confirmation by the Commission on Appointments.

** The SC finds no abuse in what GMA did. The absence of abuse is readily apparent from GMAs issuance of ad interim appointments to respondents immediately upon the recess of Congress, way before the lapse of one year.

De Castro v. JBC Facts: This case is based on multiple cases field with dealt with the controversy that has arisen from the forthcoming compulsory requirement of Chief Justice Puno on May 17, 2010 or seven days after the presidential election. On December 22, 2009, Congressman Matias V. Defensor, an ex officio member of the JBC, addressed a letter to the JBC, requesting that the process for nominations to the office of the Chief Justice be commenced immediately.

In its January 18, 2010 meeting en banc, the JBC passed a resolution which stated that they have unanimously agreed to start the process of filling up the position of Chief Justice to be vacated on May 17, 2010 upon the retirement of the incumbent Chief Justice.

As a result, the JBC opened the position of Chief Justice for application or recommendation, and published for that purpose its announcement in the Philippine Daily Inquirer and the Philippine Star.

In its meeting of February 8, 2010, the JBC resolved to proceed to the next step of announcing the names of the following candidates to invite to the public to file their sworn complaint, written report, or opposition, if any, not later than February 22, 2010.

Although it has already begun the process for the filling of the position of Chief Justice Puno in accordance with its rules, the JBC is not yet decided on when to submit to the President its list of nominees for the position due to the controversy in this case being unresolved.

The compiled cases which led to this case and the petitions of intervenors called for either the prohibition of the JBC to pass the shortlist, mandamus for the JBC to pass the shortlist, or that the act of appointing the next Chief Justice by GMA is a midnight appointment.

A precedent frequently cited by the parties is the In Re Appointments Dated March 30, 1998 of Hon. Mateo A. Valenzuela and Hon. Placido B. Vallarta as Judges of the RTC of Branch 62, Bago City and of Branch 24, Cabanatuan City, respectively, shortly

referred to here as the Valenzuela case, by which the Court held that Section 15, Article VII prohibited the exercise by the President of the power to appoint to judicial positions during the period therein fixed.

ISSUES

W/N the petitioners have legal standing?

W/N there is justiciable controversy that is ripe for judicial determination?

W/N the incumbent President appoint the next Chief Justice?

W/N mandamus and prohibition will lie to compel the submission of the shortlist of nominees by the JBC?

RULING

Petitioners have legal standing because such requirement for this case was waived by the Court.

Legal standing is a peculiar concept in constitutional law because in some cases, suits are not brought by parties who have been personally injured by the operation of a law or any other government act but by concerned citizens, taxpayers or voters who actually sue in the public interest. But even if, strictly speaking, the petitioners are not covered by the definition, it is still within the wide discretion of the Court to waive the requirement and so remove the impediment to its addressing and resolving the serious constitutional questions raised.

There is a justiciable issue

We hold that the petitions set forth an actual case or controversy that is ripe for judicial determination. The reality is that the JBC already commenced the proceedings for the selection of the nominees to be included in a short list to be submitted to the President for consideration of which of them will succeed Chief Justice Puno as the next Chief Justice. Although the position is not yet vacant, the fact that the JBC began the process of nomination pursuant to its rules and practices, although it has yet to decide whether to submit the list of nominees to the incumbent outgoing President or to the next President, makes the situation ripe for judicial determination, because the next steps are the public interview of the candidates, the preparation of the short list of candidates, and the interview of constitutional experts, as may be needed.

The resolution of the controversy will surely settle with finality the nagging questions that are preventing the JBC from moving on with the process that it already began, or that are reasons persuading the JBC to desist from the rest of the process.

PROHIBITION UNDER SECTION 15, ARTICLE VII DOES NOT APPLY TO APPOINTMENTS TO FILL A VACANCY IN THE SUPREME COURT OR TO OTHER APPOINTMENST TO THE JUDICIARY.

Two constitutional provisions seemingly in conflict:

The first, Section 15, Article VII (Executive Department), provides:

Section 15. Two months immediately before the next presidential elections and up to the end of his term, a President or Acting President shall not make appointments, except temporary appointments to executive positions when continued vacancies therein will prejudice public service or endanger public safety.

The other, Section 4 (1), Article VIII (Judicial Department), states:

Section 4. (1). The Supreme Court shall be composed of a Chief Justice and fourteen Associate Justices. It may sit en banc or in its discretion, in division of three, five, or seven Members. Any vacancy shall be filled within ninety days from the occurrence thereof.

Justification of the Supreme Court:

First. The records of the deliberations of the Constitutional Commission reveal that the framers devoted time to meticulously drafting, styling, and arranging the Constitution. Such meticulousness indicates that the organization and arrangement of the provisions of the Constitution were not arbitrarily or whimsically done by the framers, but purposely made to reflect their intention and manifest their vision of what the Constitution should contain.

The Constitution consists of 18 Articles, three of which embody the allocation of the awesome powers of government among the three great departments, the Legislative (Article VI), the Executive (Article VII), and the Judicial Departments (Article VIII). The arrangement was a true recognition of the principle of separation of powers that underlies the political structure

As can be seen, Article VII is devoted to the Executive Department, and, among others, it lists the powers vested by the Constitution in the President. The presidential power of appointment is dealt with in Sections 14, 15 and 16 of the Article.

Had the framers intended to extend the prohibition contained in Section 15, Article VII to the appointment of Members of the Supreme Court, they could have explicitly done so. They could not have ignored the meticulous ordering of the provisions. They would have easily and surely written the prohibition made explicit in Section 15, Article VII as being equally applicable to the appointment of Members of the Supreme Court in Article VIII itself, most likely in Section 4 (1), Article VIII.

Although Valenzuela came to hold that the prohibition covered even judicial appointments, it cannot be disputed that the Valenzuela dictum did not firmly rest on the deliberations of the Constitutional Commission.

Moreover, the usage in Section 4(1), Article VIII of the word shall an imperative, operating to impose a duty that may be enforced should not be disregarded. Thereby, Sections 4(1) imposes on the President the imperative duty to make an appointment of a Member of the Supreme Court within 90 days from the occurrence of the vacancy. The failure by the President to do so will be a clear disobedience to the Constitution.

The 90-day limitation fixed in Section 4(1), Article VIII for the President to fill the vacancy in the Supreme Court was undoubtedly a special provision to establish a definite mandate for the President as the appointing power, and cannot be defeated by mere judicial interpretation in Valenzuela to the effect that Section 15, Article VII prevailed because it was couched in stronger negative language.

Second. Section 15, Article VII does not apply as well to all other appointments in the Judiciary.

There is no question that one of the reasons underlying the adoption of Section 15 as part of Article VII was to eliminate midnight appointments from being made by an outgoing Chief Executive. Given the background and rationale for the prohibition in Section 15, Article VII, we have no doubt that the Constitutional Commission confined the prohibition to appointments made in the Executive Department. The framers did not need to extend the prohibition to appointments in the Judiciary, because their establishment of the JBC and their subjecting the nomination and screening of candidates for judicial positions to the unhurried and deliberate prior process of the JBC ensured that there would no longer be midnight appointments to the Judiciary. Indeed, the creation of the JBC was precisely intended to de-politicize the Judiciary by doing away with the intervention of the Commission on Appointments.

Third. As earlier stated, the non-applicability of Section 15, Article VII to appointments in the Judiciary was confirmed by then Senior Associate Justice Regalado to the JBC itself when it met on March 9, 1998 to discuss the question raised by some sectors about the constitutionality of xxx appointments to the Court of Appeals in light of the forthcoming presidential elections. He assured that on the basis of the (Constitutional) Commissions records, the election ban had no application to appointments to the Court of Appeals. This confirmation was accepted by the JBC, which then submitted to the President for consideration the nominations for the eight vacancies in the Court of Appeals.

Fourth. Of the 23 sections in Article VII, three (i.e., Section 14, Section15, and Section 16) concern the appointing powers of the President.

Section 14, Section 15, and Section 16 are obviously of the same character, in that they affect the power of the President to appoint. The fact that Section 14 and Section 16 refer only to appointments within the Executive Department renders conclusive that Section 15 also applies only to the Executive Department. This conclusion is consistent with the rule that every part of the statute must be interpreted with reference to the context, i.e. that every part must be considered together with the other parts, and kept subservient to the general intent of the whole enactment.

Fifth. To hold like the Court did in Valenzuela that Section 15 extends to appointments to the Judiciary further undermines the intent of the Constitution of ensuring the independence of the Judicial Department from the Executive and Legislative Departments. Such a holding will tie the Judiciary and the Supreme Court to the fortunes or misfortunes of political leaders vying for the Presidency in a presidential election. Consequently, the wisdom of having the new President, instead of the current incumbent President, appoint the next Chief Justice is itself suspect, and cannot ensure judicial independence, because the appointee can also become beholden to the appointing authority. In contrast, the appointment by the incumbent President does not run the same risk of compromising judicial independence, precisely because her term will end by June 30, 2010.

Sixth. The argument has been raised to the effect that there will be no need for the incumbent President to appoint during the prohibition period the successor of Chief Justice Puno within the context of Section 4 (1), Article VIII, because anyway there will still be about 45 days of the 90 days mandated in Section 4(1), Article VIII remaining.

The argument is flawed, because it is focused only on the coming vacancy occurring from Chief Justice Punos retirement by May 17, 2010. It ignores the need to apply Section 4(1) to every situation of a vacancy in the Supreme Court.

Section 4 (3), Article VII requires the regular elections to be held on the second Monday of May, letting the elections fall on May 8, at the earliest, or May 14, at the latest. If the regular presidential elections are held on May 8, the period of the prohibition is 115 days. If such elections are held on May 14, the period of the prohibition is 109 days. Either period of the prohibition is longer than the full mandatory 90-day period to fill the vacancy in the Supreme Court. The result is that there are at least 19 occasions (i.e., the difference between the shortest possible period of the ban of 109 days and the 90-day mandatory period for appointments) in which the outgoing President would be in no position to comply with the constitutional duty to fill up a vacancy in the Supreme Court. It is safe to assume that the framers of the Constitution could not have intended such an absurdity.

Seventh. As a matter of fact, in an extreme case, we can even raise a doubt on whether a JBC list is necessary at all for the President any President to appoint a Chief Justice if the appointee is to come from the ranks of the sitting justices of the Supreme Court.

Sec. 9, Article VIII says:

xxx. The Members of the Supreme Court xxx shall be appointed by the President from a list of at least three nominees prepared by the Judicial and Bar Council for any vacancy. Such appointments need no confirmation.

xxx

The provision clearly refers to an appointee coming into the Supreme Court from the outside, that is, a non-member of the Court aspiring to become one. It speaks of candidates for the Supreme Court, not of those who are already members or sitting justices of the Court, all of whom have previously been vetted by the JBC.

WRIT OF MANDAMUS DOES NOT LIE AGAINST THE JBC

Mandamus shall issue when any tribunal, corporation, board, officer or person unlawfully neglects the performance of an act that the law specifically enjoins as a duty resulting from an office, trust, or station. It is proper when the act against which it is directed is one addressed to the discretion of the tribunal or officer. Mandamus is not available to direct the exercise of a judgment or discretion in a particular way.

For mandamus to lie, the following requisites must be complied with: (a) the plaintiff has a clear legal right to the act demanded; (b) it must be the duty of the defendant to perform the act, because it is mandated by law; (c) the defendant unlawfully neglects the performance of the duty enjoined by law; (d) the act to be performed is ministerial, not discretionary; and (e) there is no appeal or any other plain, speedy and adequate remedy in the ordinary course of law.

G.R. No. L-22754: Ruben Villaluz vs Calixto Zaldivar

Control Power Removal Power Appointees

Villaluz was appointed as the Administrator of the Motor Vehicles Office in 1958. In 1960, Congressman Roces alleged that Villaluz was an ineffective leader and had caused losses to the government. He indorsed the removal of Villaluz. The Exec Sec suspended Villaluz and ordered a committee to investigate the matter. After investigation, it was recommended that she be removed. The president then issued an AO removing Villaluz from his post. Villaluz averred that the president has no jurisdiction to remove him.

ISSUE: Whether or not Villaluz is under the jurisdiction of the President to be removed considering that he is an appointee of the president.

HELD: The President of the Philippines has jurisdiction to investigate and remove him since he is a presidential appointee who belongs to the non-competitive or unclassified service under Sec 5 of RA 2260; being a presidential appointee, Villaluz belongs to the non-competitive or unclassified service of the government and as such he can only be investigated and removed from office after due hearing by the President of the Philippines under the principle that "the power to remove is inherent in the power to appoint" . There is some point in the argument that the power of control of the President may extend to the power to investigate, suspend or remove officers and employees who belong to the executive department if they are presidential appointees or do not belong to the classified service for such can be justified under the principle that the power to remove is inherent in the power to appoint but not with regard to those officers or employees who belong to the classified service for as to them that inherent power cannot be exercised. This is in line with the provision of our Constitution which says that `the Congress may by law vest the appointment of the inferior officers, in the President alone, in the courts, or in heads of department.

Citation: Ang-Angco v. Castillo, No.L-17169, SUPREME COURT OF THE REPUBLIC OF THE PHILIPPINES, 9 SCRA 619, February 16, 1960, Argued, November 30, 1963, Decided.

Facts: The Pepsi-Cola Co. requested for the withdrawal of pepsi-cola concentrates which were not covered by any Central Bank release certificate. Its counsels approached Collector of Customs AngAngco to secure the immediate release of the concentrates, but advised the counsel to secure the release certificate from the NoDollar Import Office. The Non-Dollar Import Office wrote a letter to Ang-Angco which stated that his office had no objection to the release of the concentrates but could not take action on the request as it was not in their jurisdiction. Ang-Angco telephoned the Secretary of Finance who expressed his approval of the release on the basis of said certificate. Collector Ang-Angco finally released the concentrates. When Commissioner of Customs learned of the release he filed an administrative complaint against Collector of Customs Ang-Angco. For three years Ang-Angco had been discharging the duties of his office. Then, Executive Secretary Castillo, by authority of the President, rendered his judgment against the petitioner.

Issue: Whether the President is empowered to remove officers and employees in the classified civil service.

Previous History: Secretary Castillo asserted that the President virtue of his power of control over all executive departments, bureaus and offices, can take direct action and dispose of the administrative case in subordinate officers of the executive branch of the government.

Holding: The President does not have the power to remove officers or employees in the classified civil service.

Reasoning: It is clear that under the present provision of the Civil Service Act of 1959, the case of petitioner comes under the exclusive jurisdiction of the Commissioner of Civil Service, and having been deprived of the procedure laid down in connection with the investigation and disposition of his case, it may be said that he has been deprived of due process as guaranteed by said law.

The Power of control of the President may extend to the Power to investigate, suspend or remove officers and employees who belong to the executive department

if they are presidential appointees but not with regard to those officers or employees who belong to the classified service for as to them that inherent power cannot be exercised.

This is in line with the provision of our Constitution which says that "the Congress may by law vest the appointment of the inferior officers, in the President alone, in the courts, or in heads of department" (Article VII, Section 10 [3], Constitution). With regard to these officers whose appointments are vested on heads of departments, Congress has provided by law for a procedure for their removal precisely in view of this constitutional authority. One such law is the Civil Service Act of 1959.

Significance: It well established in this case that it is contrary to law to take direct action on the administrative case of an employee under classified service even with the authority of the President without submitting the case to the Commissioner of Civil Service.

Case Digest on Pimentel v. Aguirre G.R. No. 132988 (July 19, 2000) November 10, 2010

FACTS: This is a petition for certiorari and prohibition seeking to annul Section 1 of Administrative Order No. 372, issued by the President, insofar as it requires local government units to reduce their expenditures by 25% of their authorized regular appropriations for non-personal services and to enjoin respondents from implementing Section 4 of the Order, which withholds a portion of their internal revenue allotments. HELD: Section 1 of the AO does not violate local fiscal autonomy. Local fiscal autonomy does not rule out any manner of national government intervention by way of supervision, in order to ensure that local programs, fiscal and otherwise, are consistent with national goals. AO 372 is merely directory and has been issued by the President consistent with his powers of supervision over local governments. A directory order cannot be characterized as an exercise of the power of control. The AO is intended only to advise all government agencies and instrumentalities to undertake cost-reduction measures that will help maintain economic stability in the country. It does not contain any sanction in case of noncompliance. The Local Government Code also allows the President to interfere in local fiscal matters, provided that certain requisites are met: (1) an unmanaged public sector deficit of the national government; (2) consultations with the presiding officers of the Senate and the House of Representatives and the presidents of the various local leagues; (3) the corresponding recommendation of the secretaries of the Department of Finance, Interior and Local Government, and Budget and Management; and (4) any adjustment in the allotment shall in no case be less than 30% of the collection of national internal revenue taxes of the third fiscal year preceding the current one. Section 4 of AO 372 cannot be upheld. A basic feature of local fiscal autonomy is the automatic release of the shares of LGUs in the national internal revenue. This is mandated by the Constitution and the Local Government Code. Section 4 which orders the withholding of 10% of the LGUs IRA clearly contravenes the Constitution and the law.

Gudani vs. Senga G.R. No. 170165, Aug. 15, 2006

The ability of the President to prevent military officers from testifying before Congress does not turn on executive privilege, but on the Chief Executives power as commander-in-chief to control the actions and speech of members of the armed forces. The Presidents prerogatives as commander-in-chief are not hampered by the same limitations as in executive privilege.

FACTS:

On Sept. 22, 2005, Sen. Biazon invited several senior officers of the AFP, including Gen. Gudani, to appear at a public hearing before the Senate Committee on National Defense and Security concerning the conduct of the 2004 elections wherein allegations of massive cheating and the Hello Garci tapes emerged. AFP Chief of Staff Gen. Senga issued a Memorandum, prohibiting Gen. Gudani, Col. Balutan and company from appearing before the Senate Committee without Presidential approval. Nevertheless, Gen. Gudani and Col. Balutan testified before said Committee, prompting Gen. Senga to order them subjected to General Court Martial proceedings for willfully violating an order of a superior officer. In the meantime, President Arroyo issued EO 464, which was subsequently declared unconstitutional.

ISSUE:

Whether or not the President can prevent military officers from testifying at a legislative inquiry

RULING:

We hold that the President has constitutional authority to do so, by virtue of her power as commander-in-chief, and that as a consequence a military officer who defies such injunction is liable under military justice. At the same time, we also hold that any chamber of Congress which seeks the appearance before it of a military officer against the consent of the President has adequate remedies under law to compel such attendance. Any military official whom Congress summons to testify before it may be compelled to do so by the President. If the President is not so inclined, the President may be commanded by judicial order to compel the attendance of the military officer. Final judicial orders have the force of the law of the land which the President has the duty to faithfully execute.

Ability of President to prevent military officers from testifying before Congress is based on Commander-in-chief powers

As earlier noted, we ruled in Senate that the President may not issue a blanket requirement of prior consent on executive officials summoned by the legislature to attend a congressional hearing. In doing so, the Court recognized the considerable limitations on executive privilege, and affirmed that the privilege must be formally invoked on specified grounds. However, the ability of the President to prevent military officers from testifying before Congress does not turn on executive privilege, but on the Chief Executives power as commander-in-chief to control the actions and speech of members of the armed forces. The Presidents prerogatives as commander-in-chief are not hampered by the same limitations as in executive privilege.

RATIONALE: Our ruling that the President could, as a general rule, require military officers to seek presidential approval before appearing before Congress is based foremost on the notion that a contrary rule unduly diminishes the prerogatives of the President as commander-in-chief. Congress holds significant control over the armed forces in matters such as budget appropriations and the approval of higherrank promotions, yet it is on the President that the Constitution vests the title as commander-in-chief and all the prerogatives and functions appertaining to the position. Again, the exigencies of military discipline and the chain of command mandate that the Presidents ability to control the individual members of the armed forces be accorded the utmost respect. Where a military officer is torn between obeying the President and obeying the Senate, the Court will without hesitation affirm that the officer has to choose the President. After all, the Constitution prescribes that it is the President, and not the Senate, who is the commander-inchief of the armed forces.

Remedy is judicial relief

At the same time, the refusal of the President to allow members of the military to appear before Congress is still subject to judicial relief. The Constitution itself recognizes as one of the legislatures functions is the conduct of inquiries in aid of legislation. Inasmuch as it is ill-advised for Congress to interfere with the Presidents power as commander-in-chief, it is similarly detrimental for the President to unduly interfere with Congresss right to conduct legislative inquiries. The impasse did not come to pass in this petition, since petitioners testified anyway despite the presidential prohibition. Yet the Court is aware that with its pronouncement today that the President has the right to require prior consent from members of the armed forces, the clash may soon loom or actualize.

We believe and hold that our constitutional and legal order sanctions a modality by which members of the military may be compelled to

Prof. Randolf S. David vs. Gloria Macapagal-Arroyo G.R. No. 171396, May 3, 2006

"Take Care" Power of the President Powers of the Chief Executive The power to promulgate decrees belongs to the Legislature

FACTS:

These 7 consolidated petitions question the validity of PP 1017 (declaring a state of national emergency) and General Order No. 5 issued by President Gloria MacapagalArroyo. While the cases are pending, President Arroyo issued PP 1021, declaring that the state of national emergency has ceased to exist, thereby, in effect, lifting PP 1017.

ISSUE:

Whether or not PP 1017 and G.O. No. 5 arrogated upon the President the power to enact laws and decrees If so, whether or not PP 1017 and G.O. No. 5 are unconstitutional

HELD:

Take-Care Power

This refers to the power of the President to ensure that the laws be faithfully executed, based on Sec. 17, Art. VII: The President shall have control of all the executive departments, bureaus and offices. He shall ensure that the laws be faithfully executed.

As the Executive in whom the executive power is vested, the primary function of the President is to enforce the laws as well as to formulate policies to be embodied in existing laws. He sees to it that all laws are enforced by the officials and employees of his department. Before assuming office, he is required to take an oath or affirmation to the effect that as President of the Philippines, he will, among others, execute its laws. In the exercise of such function, the President, if needed, may employ the powers attached to his office as the Commander-in-Chief of all the armed forces of the country, including the Philippine National Police under the Department of Interior and Local Government.

The specific portion of PP 1017 questioned is the enabling clause: to enforce obedience to all the laws and to all decrees, orders and regulations promulgated by me personally or upon my direction.

Is it within the domain of President Arroyo to promulgate decrees?

The President is granted an Ordinance Power under Chap. 2, Book III of E.O. 292. President Arroyos ordinance power is limited to those issuances mentioned in the foregoing provision. She cannot issue decrees similar to those issued by Former President Marcos under PP 1081. Presidential Decrees are laws which are of the same category and binding force as statutes because they were issued by the President in the exercise of his legislative power during the period of Martial Law under the 1973 Constitution.

This Court rules that the assailed PP 1017 is unconstitutional insofar as it grants President Arroyo the authority to promulgate decrees. Legislative power is peculiarly within the province of the Legislature. Sec. 1, Art. VI categorically states that the legislative power shall be vested in the Congress of the Philippines which shall consist of a Senate and a House of Representatives. To be sure, neither Martial Law nor a state of rebellion nor a state of emergency can justify President Arroyos exercise of legislative power by issuing decrees.

But can President Arroyo enforce obedience to all decrees and laws through the military?

As this Court stated earlier, President Arroyo has no authority to enact decrees. It follows that these decrees are void and, therefore, cannot be enforced. With respect to laws, she cannot call the military to enforce or implement certain laws, such as customs laws, laws governing family and property relations, laws on obligations and contracts and the like. She can only order the military, under PP 1017, to enforce laws pertinent to its duty to suppress lawless violence.

Olaguer vs Military Commission

Habeas Corpus

In 1979, Olaguer and some others were detained by military personnel and they were placed in Camp Bagong Diwa. Logauer and his group are all civilians. They were charged with (1) unlawful possession of explosives and incendiary devices; (2) conspiracy to assassinate President and Mrs. Marcos; (3) conspiracy to assassinate cabinet members Juan Ponce Enrile, Francisco Tatad and Vicente Paterno; (4) conspiracy to assassinate Messrs. Arturo Tangco, Jose Roo and Onofre Corpus; (5) arson of nine buildings; (6) attempted murder of Messrs. Leonardo Perez, Teodoro Valencia and Generals Romeo Espino and Fabian Ver; and (7) conspiracy and proposal to commit rebellion, and inciting to rebellion. On August 19, 1980, the petitioners went to the SC and filed the instant Petition for prohibition and habeas corpus.

ISSUE: Whether or not the petition for habeas corpus be granted.

HELD: The petition for habeas corpus has become moot and academic because by the time the case reached the SC Olaguer and his companions were already released from military confinement. When the release of the persons in whose behalf the application for a writ of habeas corpus was filed is effected, the Petition for the issuance of the writ becomes moot and academic. 18 Inasmuch as the herein petitioners have been released from their confinement in military detention centers, the instant Petitions for the issuance of a writ of habeas corpus should be dismissed for having become moot and academic. But the military court created to try the case of Olaguer (and the decision it rendered) still continues to subsist.

ISSUE2: The issue is then shifted to: Whether or not a military tribunal has the jurisdiction to try civilians while the civil courts are open and functioning.

HELD: The SC nullified for lack of jurisdiction all decisions rendered by the military courts or tribunals during the period of martial law in all cases involving civilian defendants. A military commission or tribunal cannot try and exercise jurisdiction, even during the period of martial law, over civilians for offenses allegedly committed by them as long as the civil courts are open and functioning, and that any judgment rendered by such body relating to a civilian is null and void for lack of jurisdiction on the part of the military tribunal concerned.

LANSANG VS. GARCIA [42 SCRA 448; L-33964; 11 Dec 1971] Monday, February 09, 2009 Posted by Coffeeholic Writes Labels: Case Digests, Political Law

Facts: In the evening of August 21, 1971, at about 9 p.m., while the Liberal Party of the Philippines was holding a public meeting at Plaza Miranda, Manila, for the presentation of its candidates in the general elections scheduled for November 8, 1971, two hand grenades were thrown at the platform where said candidates and other persons were. Eight persons were killed and many more injured. Proclamation 889 was issued by the President suspending privilege of writ of habeas corpus stating that there is a conspiracy of rebellion and insurrection in order to forcibly seize political power. Petitions for writ of habeas corpus were filed by persons (13) who have been arrested without a warrant.

It was stated that one of the safeguards of the proclamation was that it is to be applied to persons caught in flagrante delicto. Incidentally, Proc. 889-A was issued as an amendment, inserting the word actually staging. Proc. 889-B was also issued lifting the suspension of privilege in 27 provinces, 3 sub-provinces and 26 cities. Proc. 889-C was issued restoring the suspension in 13 provinces and cities(mostly in Mindanao). Proc. 889-D further lifted the suspension in 7 provinces and 4 cities. Only 18 provinces and sub-provinces and 2 cities whose privilege was suspended. Petitioners maintained that Proclamation No. 889 did not declare the existence of actual "invasion insurrection or rebellion or imminent danger thereof, however it became moot and academic since it was amended. Petitioners further contend that public safety did not require the issuance of proclamations stating: (a) that there is no rebellion; (b) that, prior to and at the time of the suspension of the privilege, the Government was functioning normally, as were the courts; (c) that no untoward incident, confirmatory of an alleged July-August Plan, has actually taken place after August 21, 1971; (d) that the President's alleged apprehension, because of said plan, is non-existent and unjustified; and (e) that the Communist forces in the Philippines are too small and weak to jeopardize public safety to such extent as to require the suspension of the privilege of the writ of habeas corpus.

A resolution was issued by majority of the Court having tentatively arrived at a consensus that it may inquire in order to satisfy itself of the existence of the factual bases for the proclamations. Now the Court resolves after conclusive decision reached by majority.

Issues:

(1) Whether or Not the authority to decide whether the exigency has arisen requiring suspension (of the privilege of the writ of habeas corpus) belongs to the President and his decision is final and conclusive upon the courts and upon all other persons.

(2) Whether or Not public safety require the suspension of the privilege of the writ of habeas corpus decreed in Proclamation No. 889-A.

Held: The President has authority however it is subject to judicial review. Two conditions must concur for the valid exercise of the authority to suspend the privilege to the writ (a) there must be "invasion, insurrection, or rebellion" or "imminent danger thereof," and (b) "public safety" must require the suspension of the privilege. President has three (3) courses of action: (a) to call out the armed forces; (b) to suspend the privilege of the writ of habeas corpus; and (c) to place the Philippines or any part thereof under martial law. He had, already, called out the armed forces, proved inadequate. Of the two other alternatives, the suspension of the privilege is the least harsh.

Petitioners contention that CPP-NPA has no ability, is negatived by the killing of 5 mayors, 20 barrio captains and 3 chiefs of police; that there were fourteen (14) meaningful bombing incidents in the Greater Manila Area in 1970. CPP has managed to infiltrate or establish and control nine major labor organizations; has exploited the (11) major student or youth organizations; about thirty (30) mass organizations actively advancing the CPP.

CASE DIGEST ON PEOPLE v. NATIONAL [248 SCRA 122 (1995)] November 10, 2010

Facts: Nacional, Millamino, Musa, Lucer, Mirabete and Militante member of NPA charged with murder and Quirino and Joel Lagaron in Daragan Albay on Feb. 21, 1985. All except Mirabete were given conditional pardon as political prisoners. Lagarons were suspected government informers. Issue: WON Mirabete is criminally liable Held: Yes. Conviction affirmed. He was identified by witnesses. Rincopan identified him as CPP-NPA member present during pulong-pulong. There was evident premeditation as the pulong pulong decided to liquidate two and assigned roles. In People v. Talla, conspiracy is said to exist once agreed upon expressly or impliedly to commit felonythis was established by meeting before crime committed. In People v Timple, it is said the conspirators are liable for acts of others. This is reiterated in People v. Apawan, which states that all conspirators are liable, and that act of one is act of all regardless of degree of participation.

Wilfredo Torres vs Hon. Neptali Gonzales on October 28, 2011 00

Constitutional Law Pardon Not Subject to Judicial Review/Scrutiny

In 1978, Torres was convicted of estafa. In 1979, he was pardoned by the president w/ the condition that he shall not violate any penal laws again. In 1982, Torres was charged with multiple crimes of estafa. In 1986, Gonzales petitioned for the cancellation of Torres pardon. Hence, the president cancelled the pardon. Torres appealed the issue before the SC averring that the Exec Dept erred in convicting him for violating the conditions of his pardon because the estafa charges against him were not yet final and executory as they were still on appeal.

ISSUE: Whether or not conviction of a crime by final judgment of a court is necessary before Torres can be validly rearrested and recommitted for violation of the terms of his conditional pardon and accordingly to serve the balance of his original sentence.

HELD: The SC affirmed the following:

1. The grant of pardon and the determination of the terms and conditions of a conditional pardon are purely executive acts which are not subject to judicial scrutiny.

2. The determination of the occurrence of a breach of a condition of a pardon, and the proper consequences of such breach, may be either a purely executive act, not subject to judicial scrutiny under Section 64 (i) of the Revised Administrative Code; or it may be a judicial act consisting of trial for and conviction of violation of a conditional pardon under Article 159 of the Revised Penal Code. Where the President opts to proceed under Section 64 (i) of the Revised Administrative Code, no judicial pronouncement of guilt of a subsequent crime is necessary, much less conviction therefor by final judgment of a court, in order that a convict may be recommended for the violation of his conditional pardon.

3. Because due process is not semper et ubique judicial process, and because the conditionally pardoned convict had already been accorded judicial due process in his trial and conviction for the offense for which he was conditionally pardoned, Section 64 (i) of the Revised Administrative Code is not afflicted with a constitutional vice.

In proceeding against a convict who has been conditionally pardoned and who is alleged to have breached the conditions of his pardon, the Executive Department has two options: (i) to proceed against him under Section 64 (i) of the Revised Administrative Code; or (ii) to proceed against him under Article 159 of the RPC which imposes the penalty of prision correccional, minimum period, upon a convict who having been granted conditional pardon by the Chief Executive, shall violate any of the conditions of such pardon. Here, the President has chosen to proceed against the petitioner under Section 64 (i) of the Revised Administrative Code. That choice is an exercise of the Presidents executive prerogative and is not subject to judicial scrutiny.

People vs. Patriarca, Jr. G.R. No. 135457, September 29, 2000 Sunday, January 25, 2009 Posted by Coffeeholic Writes Labels: Case Digests, Political Law

Facts: Accused-appellant Jose Patriarca, Jr., a member of the NPA, was found guilty by the trial court of the crime of murder for the death of Alfredo Arevalo and was sentenced to suffer the penalty of reclusion perpetua. Accused-appellant appealed the decision of the RTC.

Accused-appellant applied for amnesty under Proclamation No. 724. His application was favorably granted by the National Amnesty Board concluding that his activities were done in pursuit of his political beliefs.

Issue: What is the effect of the grant of amnesty to the conviction of the accusedappellant?

Held: Amnesty commonly denotes a general pardon to rebels for their treason or other high political offenses, or the forgiveness which one sovereign grants to the subjects of another, who have offended, by some breach, the law of nations. Amnesty looks backward, and abolishes and puts into oblivion, the offense itself; it so overlooks and obliterates the offense with which he is charged, that the person released by amnesty stands before the law precisely as though he had committed no offense.

Paragraph 3 of Art. 89 of the Revised Penal Code provides that criminal liability is totally extinguished by amnesty, which completely extinguishes the penalty and all its effects.

The grant of amnesty serves to put an end to the appeal. Accused-appellant is acquitted of the crime of murder.

BAYAN VS. EXECUTIVE SECRETARY ERMITA [488 SCRA 226; G.R. NO. 169838; 25 APR 2006] Sunday, February 08, 2009 Posted by Coffeeholic Writes Labels: Case Digests, Political Law

Facts: Rallies of September 20, October 4, 5 and 6, 2005 is at issue. BAYANs rally was violently dispersed. 26 petitioners were injured, arrested and detained when a peaceful mass action they was preempted and violently dispersed by the police. KMU asserts that the right to peaceful assembly, are affected by Batas Pambansa No. 880 and the policy of Calibrated Preemptive Response (CPR) being followed to implement it. KMU, et al., claim that on October 4, 2005, a rally KMU co-sponsored was to be conducted at the Mendiola bridge but police blocked them along C.M. Recto and Lepanto Streets and forcibly dispersed them, causing injuries to several of their members. They further allege that on October 6, 2005, a multi-sectoral rally which KMU also co-sponsored was scheduled to proceed along Espaa Avenue in front of the UST and going towards Mendiola bridge. Police officers blocked them along Morayta Street and prevented them from proceeding further. They were then forcibly dispersed, causing injuries on one of them. Three other rallyists were arrested.

All petitioners assail Batas Pambansa No. 880 The Public Assembly Act of 1985, some of them in toto and others only Sections 4, 5, 6, 12, 13(a), and 14(a), as well as the policy of CPR. They seek to stop violent dispersals of rallies under the no permit, no rally policy and the CPR policy announced on Sept. 21, 2005.

Petitioners Bayan, et al., contend that BP 880 is clearly a violation of the Constitution and the International Covenant on Civil and Political Rights and other human rights treaties of which the Philippines is a signatory.

They argue that B.P. No. 880 requires a permit before one can stage a public assembly regardless of the presence or absence of a clear and present danger. It also curtails the choice of venue and is thus repugnant to the freedom of expression clause as the time and place of a public assembly form part of the message for which the expression is sought.

Petitioners Jess del Prado, et al., in turn, argue that B.P. No. 880 is unconstitutional as it is a curtailment of the right to peacefully assemble and petition for redress of

grievances because it puts a condition for the valid exercise of that right. It also characterizes public assemblies without a permit as illegal and penalizes them and allows their dispersal. Thus, its provisions are not mere regulations but are actually prohibitions. Regarding the CPR policy, it is void for being an ultra vires act that alters the standard of maximum tolerance set forth in B.P. No. 880, aside from being void for being vague and for lack of publication.

KMU, et al., argue that the Constitution sets no limits on the right to assembly and therefore B.P. No. 880 cannot put the prior requirement of securing a permit. And even assuming that the legislature can set limits to this right, the limits provided are unreasonable: First, allowing the Mayor to deny the permit on clear and convincing evidence of a clear and present danger is too comprehensive. Second, the five-day requirement to apply for a permit is too long as certain events require instant public assembly, otherwise interest on the issue would possibly wane.As to the CPR policy, they argue that it is preemptive, that the government takes action even before the rallyists can perform their act, and that no law, ordinance or executive order supports the policy. Furthermore, it contravenes the maximum tolerance policy of B.P. No. 880 and violates the Constitution as it causes a chilling effect on the exercise by the people of the right to peaceably assemble.

Respondents argued that petitioners have no standing. BP 880 entails traffic rerouting to prevent grave public inconvenience and serious or undue interference in the free flow of commerce and trade. It is content-neutral regulation of the time, place and manner of holding public assemblies. According to Atienza RA. 7160 gives the Mayor power to deny a permit independently of B.P. No. 880. and that the permit is for the use of a public place and not for the exercise of rights; and that B.P. No. 880 is not a content-based regulation because it covers all rallies.

Issue: Whether or Not BP 880 and the CPR Policy unconstitutional.

Held: No question as to standing. Their right as citizens to engage in peaceful assembly and exercise the right of petition, as guaranteed by the Constitution, is directly affected by B.P. No. 880. B.P. 880 is not an absolute ban of public assemblies but a restriction that simply regulates the time, place and manner of the assemblies. It refers to all kinds of public assemblies that would use public places. The reference to lawful cause does not make it content-based because assemblies really have to

be for lawful causes, otherwise they would not be peaceable and entitled to protection. Maximum tolerance1 is for the protection and benefit of all rallyists and is independent of the content of the expressions in the rally. There is, likewise, no prior restraint, since the content of the speech is not relevant to the regulation.

The so-called calibrated preemptive response policy has no place in our legal firmament and must be struck down as a darkness that shrouds freedom. It merely confuses our people and is used by some police agents to justify abuses. Insofar as it would purport to differ from or be in lieu of maximum tolerance, this was declared null and void.

The Secretary of the Interior and Local Governments, are DIRECTED to take all necessary steps for the immediate compliance with Section 15 of Batas Pambansa No. 880 through the establishment or designation of at least one suitable freedom park or plaza in every city and municipality of the country. After thirty (30) days from the finality of this Decision, subject to the giving of advance notices, no prior permit shall be required to exercise the right to peaceably assemble and petition in the public parks or plazas of a city or municipality that has not yet complied with Section 15 of the law.

Potrebbero piacerti anche